Search type Search syntax
Tags [tag]
Exact "words here"
Author user:1234
user:me (yours)
Score score:3 (3+)
score:0 (none)
Answers answers:3 (3+)
answers:0 (none)
isaccepted:yes
hasaccepted:no
inquestion:1234
Views views:250
Code code:"if (foo != bar)"
Sections title:apples
body:"apples oranges"
URL url:"*.example.com"
Saves in:saves
Status closed:yes
duplicate:no
migrated:no
wiki:no
Types is:question
is:answer
Exclude -[tag]
-apples
For more details on advanced search visit our help page
Results tagged with
Search options questions only not deleted user 3324

Algebraic and geometric theory of quadratic forms and symmetric bilinear forms, e.g., values attained by quadratic forms, isotropic subspaces, the Witt ring, invariants of quadratic forms, the discriminant and Clifford algebra of a quadratic form, Pfister forms, automorphisms of quadratic forms.

7 votes
2 answers
711 views

Mass of spinor genus, positive integral quadratic forms

There seems to be general opinion that, for positive integral quadratic forms in at least three variables, spinor genera in the same genus all have the same mass (not representation measures of some n …
Will Jagy's user avatar
  • 25.2k
3 votes
1 answer
394 views

Faux Mordell equation and positive binary quadratic forms

This is about the frequency of integral solutions to $$ b^2 - 4 a^3 = \Delta, $$ when $\Delta < 0$ is a discriminant of positive binary quadratic forms such that the class number is divisible by 3. …
Will Jagy's user avatar
  • 25.2k
1 vote
1 answer
184 views

probably Lagrange or Legendre, Pell variant

Evidently Legendre showed that, for positive primes, if $p \equiv 3 \pmod 8$ there is an integral solution to $x^2 - p y^2 = -2.$ Next, if $q \equiv 7 \pmod 8$ there is an integral solution to $x^2 - …
Will Jagy's user avatar
  • 25.2k
4 votes
1 answer
169 views

optimal bound in diophantine representation question

Given that, with integers $t \geq 1$ and $q \geq 3,$ there are solutions to $$ x^2 - q x y + y^2 = - t q $$ with integers $x,y \geq 1,$ I was able to show that $$ q \leq 1 + \frac{324}{25} t^2. $$ …
Will Jagy's user avatar
  • 25.2k
1 vote
0 answers
183 views

Watson Transformation Squared reference request

See http://www.numbertheory.org/obituaries/OTHERS/watson.html George Leo Watson (1909-1988) wrote in a conversational manner, it is difficult to see when he switches from the trivial to the incredibl …
Will Jagy's user avatar
  • 25.2k
5 votes
2 answers
293 views

Matrix version of number theoretic integral lattice claim

I know this not, everybody else seems to. This is from page 215 of Cassels, Rational Quadratic Forms, formula 4.1, or SPLAG, page 389 formula (36). quote: If $f$ and $g$ are forms of determinant …
Will Jagy's user avatar
  • 25.2k
7 votes
0 answers
437 views

When is the set of numbers represented by certain quaternary quadratic forms completely mult...

Expired by this question A quadratic form represents all primes except for the primes 2 and 11. I would like to know some simple sufficient conditions for when the set of numbers integrally represe …
Will Jagy's user avatar
  • 25.2k
9 votes
2 answers
2k views

Does a positive binary quadratic form represent a set of primes possessing a natural density

In his answer to my question The Green-Tao theorem and positive binary quadratic forms Kevin Ventullo answers my initial question in the affirmative. What remains is the title questio …
Will Jagy's user avatar
  • 25.2k
11 votes
1 answer
814 views

Primes $ 1 + x^2 + y^2$

EDIT, Saturday 11:32 am, March 24: a complete answer for $4 + x^2 + y^2$ and generally $4 m^2 + x^2 + y^2$ for fixed $m$ is given in Friedlander and Iwaniec page 282, Theorem 14.8. We might also expec …
Will Jagy's user avatar
  • 25.2k
8 votes
3 answers
1k views

Integral orthogonal group for indefinite ternary quadratic form

I have the indefinite quadratic form $q(x,y,z) = 19 x^2 + 5 y^2 - z^2.$ It's not my fault. I find, on reflection, that I have no idea how to describe the orthogonal group of this over the integers. Th …
Will Jagy's user avatar
  • 25.2k
7 votes
3 answers
1k views

Fricke Klein method for isotropic ternary quadratic forms

Preface: the most natural way to take one isotropic vector for an indefinite quadratic form and find others is to use stereographic projection. This gives a parametrization in the same $n$ variables a …
Will Jagy's user avatar
  • 25.2k
11 votes
2 answers
995 views

Positive primes represented by indefinite binary quadratic form

Neil Sloane asked me about commands in computer languages to find the (positive) primes represented by indefinite binary quadratic forms. So I wrote something in C++ that works. This is for the OEIS, …
Will Jagy's user avatar
  • 25.2k
11 votes
1 answer
714 views

Positive ternary quadratic forms in the same genus that represent the same numbers

There are three genera of positive, integral, ternary quadratic forms in which both forms (classes...) are regular, so the paired forms represent the same numbers. These pairs (complete genera) are: …
Will Jagy's user avatar
  • 25.2k
7 votes
2 answers
630 views

Verifying an example in the Geometry of Numbers and Quadratic Forms

In answer to Pete L. Clark's question Must a ring which admits a Euclidean quadratic form be Euclidean? on Euclidean quadratic forms, I gave an example in seven variables, repeated below. Pete's Eucl …
Will Jagy's user avatar
  • 25.2k
6 votes
1 answer
353 views

Erdos Kac for imaginary class number

In answer to A coverage question Cam mentions an article by SOUND. I have been running a computer program for THIS and would like to know if there are a reasonable average and standard deviation for …
Will Jagy's user avatar
  • 25.2k

15 30 50 per page